Sie sind auf Seite 1von 33

Part 2 : 01/14/19 11:29:25

Question 1 - CMA 1289 1.22 - Short-term Bank Loans and Other S-T Financing

The principal advantage of using commercial paper as a short-term financing instrument is that it

A. Is readily available to almost all companies.


B. Is usually cheaper than a commercial bank loan.
C. Can be purchased without commission costs.
D. Offers security, i.e., collateral, to the lender.

Question 2 - ICMA 10.P2.183 - Short-term Bank Loans and Other S-T Financing

The effective annual interest rate to the borrower of a $100,000 one-year loan with a stated rate of 7% and a 20%
compensating balance is

A. 8.4%.
B. 7.0%.
C. 13.0%.
D. 8.75%.

Question 3 - ICMA 10.P2.150 - Short-term Bank Loans and Other S-T Financing

The Duoplan Company is determining the most appropriate source of short-term funding. Trade credit terms from
suppliers are 2/30, net 90. The rate for borrowing at the bank is 12%. The company has also been approached by an
investment banker offering to issue Duoplan's commercial paper. The commercial paper would be issued quarterly in
increments of $9.1 million with net proceeds of $8.8 million. Which option should the firm select?

A. Bank borrowing, because it provides the lowest cost of funds.


B. The costs are so similar that the decision is a matter of convenience.
C. Commercial paper, because it provides the lowest cost of funds.
D. The trade discount, because it provides the lowest cost of funds.

Question 4 - CMA Sample Q1.7 - Short-term Bank Loans and Other S-T Financing

On January 1, Scott Corporation received a $300,000 line of credit at an interest rate of 12% from Main Street Bank
and drew down the entire amount on February 1. The line of credit agreement requires that an amount equal to 15% of
the loan be deposited into a compensating balance account. What is the effective annual cost of credit for this loan
arrangement?

A. 14.12%
B. 11.00%
C. 12.00%
D. 12.94%

Question 5 - ICMA 10.P2.171 - Short-term Bank Loans and Other S-T Financing

Burke Industries has a revolving credit arrangement with its bank which specifies that Burke can borrow up to $5
million at an annual interest rate of 9% payable monthly. In addition, Burke must pay a commitment fee of 0.25% per

(c) HOCK international, page 1


Part 2 : 01/14/19 11:29:25

million at an annual interest rate of 9% payable monthly. In addition, Burke must pay a commitment fee of 0.25% per
month on the unused portion of the line, payable monthly. Burke expects to have a $2 million cash balance and no
borrowings against this line of credit on April 1, net cash inflows of $2 million in April, net outflows of $7 million in May,
and net inflows of $4 million in June. If all cash flows occur at the end of the month, approximately how much will Burke
pay to the bank during the second quarter related to this revolving credit arrangement?

A. $52,600.
B. $60,200.
C. $47,700.
D. $62,500.

Question 6 - CMA 694 1.21 - Short-term Bank Loans and Other S-T Financing

A company obtained a short-term bank loan of $500,000 at an annual interest rate of 8%. As a condition of the loan,
the company is required to maintain a compensating balance of $100,000 in its checking account. The checking
account earns interest at an annual rate of 3%. Ordinarily, the company maintains a balance of $50,000 in its account
for transaction purposes. What is the effective interest rate of the loan?

A. 8.22%
B. 9.25%
C. 7.77%
D. 8.56%

Question 7 - CMA 1290 1.28 - Short-term Bank Loans and Other S-T Financing

Corbin, Inc. can issue 3-month commercial paper with a face value of $1,000,000 for $980,000. Transaction costs will
be $1,200. The effective annualized percentage cost of the financing, based on a 360-day year, will be

A. 8.17%.
B. 8.66%.
C. 8.00%.
D. 2.00%.

Question 8 - CMA 1296 1.17 - Short-term Bank Loans and Other S-T Financing

The Frame Supply Company has just acquired a large account and needs to increase its working capital by $100,000.
The controller of the company has identified a number of sources. One of them is:
Borrow $125,000 from a bank on a discount basis for one year at 20%. No compensating balance would be
required.
Assume a 360-day year in all of your calculations.

The cost of this alternative is:

A. 50.0%
B. 40.0%
C. 25.0%
D. 20.0%

(c) HOCK
Question 9 - CMA 1294 1.23 - Short-term Bank Loansinternational,
and Other S-Tpage 2
Financing

If a firm borrows $500,000 at 10% and is required to maintain $50,000 as a minimum compensating balance at the
bank, what is the effective interest rate on the loan?
Part 2 : 01/14/19 11:29:25

Question 9 - CMA 1294 1.23 - Short-term Bank Loans and Other S-T Financing

If a firm borrows $500,000 at 10% and is required to maintain $50,000 as a minimum compensating balance at the
bank, what is the effective interest rate on the loan?

A. 12.2%
B. 10.0%
C. 11.1%
D. 9.1%

Question 10 - CMA 1295 1.10 - Short-term Bank Loans and Other S-T Financing

The Dixon Corporation has an outstanding 1-year bank loan of $300,000 at a stated interest rate of 8%. In addition,
Dixon is required to maintain a 20% compensating balance in its checking account. Assuming the company would
normally maintain a zero balance in its checking account, the effective interest rate on the loan is

A. 8.0%
B. 6.4%
C. 10.0%
D. 20.0%

Question 11 - CMA 696 1.11 - Short-term Bank Loans and Other S-T Financing

A company obtained a short-term bank loan of $250,000 at an annual interest rate of 6%. As a condition of the loan,
the company is required to maintain a compensating balance of $50,000 in its checking account. The company's
checking account earns interest at an annual rate of 2%. Ordinarily, the company maintains a balance of $25,000 in its
checking account for transaction purposes. What is the effective interest rate of the loan?

A. 6.66%
B. 7.00%
C. 6.44%
D. 5.80%

Question 12 - CMA 1296 1.16 - Short-term Bank Loans and Other S-T Financing

The Frame Supply Company has just acquired a large account and needs to increase its working capital by $100,000.
The controller of the company has identified a number of sources. One of them is:

Issue $110,000 of 6-month commercial paper to net $100,000. (New paper would be issued every 6 months.)

Assume a 360-day year in all of your calculations.

The cost of this alternative is

A. 20.0%
B. 9.1%
C. 10.0%
D. 18.2%

(c) HOCK international, page 3

Question 13 - ICMA 10.P2.184 - Short-term Bank Loans and Other S-T Financing
Part 2 : 01/14/19 11:29:25

Question 13 - ICMA 10.P2.184 - Short-term Bank Loans and Other S-T Financing

Todd Manufacturing Company needs a $100 million loan for one year. Todd’s banker has presented two alternatives
as follows:

Option #1 - $100 million loan with a stated interest rate of 10.25%. No compensating balance required.

Option #2 - $100 million loan with a stated interest rate of 10.00%. Non-interest bearing compensating balance
required.

Which of the following compensating balances, withheld from the loan proceeds, would result in Option #2 having an
effective interest rate equal to the 10.25% rate of Option #1?

A. $2,500,000.
B. $10,250,000.
C. $250,000.
D. $2,440,000.

Question 14 - CMA 694 1.29 - Short-term Bank Loans and Other S-T Financing

A firm that often factors its accounts receivable has an agreement with its finance company that requires the firm to
maintain a 6% reserve and charges 1% commission on the amount of receivables. The net proceeds would be further
reduced by an annual interest charge of 10% on the monies advanced. Assuming a 360-day year, what amount of
cash (rounded to the nearest dollar) will the firm receive from the finance company at the time a $100,000 account that
is due in 90 days is turned over to the finance company assuming the firm withdraws the full amount of cash available
immediately?

A. $93,000
B. $90,675
C. $90,000
D. $83,700

Question 15 - CMA 696 1.30 - Short-term Bank Loans and Other S-T Financing

A company enters into an agreement with a firm that will factor the company's accounts receivable. The factor agrees
to buy the company's receivables, which average $100,000 per month and have an average collection period of 30
days. The factor will advance up to 80% of the face value of receivables at an annual rate of 10% and charge a fee of
2% on all receivables purchased. The controller of the company estimates that the company would save $18,000 in
collection expenses over the year. Fees and interest are not deducted in advance. Assuming a 360-day year, what is
the annual cost of financing?

A. 10.0%
B. 14.0%
C. 17.5%
D. 12.0%

Question 16 - CMA 1296 1.15 - Short-term Bank Loans and Other S-T Financing

The Frame Supply Company has just acquired (c) HOCK


a largeinternational, page to
account and needs 4 increase its working capital by $100,000.
The controller of the company has identified a number of sources. One of them is:
Borrow $110,000 from a bank at 12% interest. A 9% compensating balance would be required.
Assume a 360-day year in all of your calculations.
Part 2 : 01/14/19 11:29:25

Question 16 - CMA 1296 1.15 - Short-term Bank Loans and Other S-T Financing

The Frame Supply Company has just acquired a large account and needs to increase its working capital by $100,000.
The controller of the company has identified a number of sources. One of them is:
Borrow $110,000 from a bank at 12% interest. A 9% compensating balance would be required.
Assume a 360-day year in all of your calculations.

The cost of this alternative is:

A. 13.2%
B. 12.0%
C. 9.0%
D. 21.0%

Question 17 - ICMA 10.P2.186 - Short-term Bank Loans and Other S-T Financing

What is the effective annual interest rate for a one-year $100 million loan with a stated interest rate of 8.00%, if the
lending bank requires a non-interest bearing compensating balance in the amount of $5 million?

A. 13.00%.
B. 8.42%.
C. 8.00%.
D. 7.62%.

Question 18 - CMA 1295 1.11 - Short-term Bank Loans and Other S-T Financing

Elan Corporation is considering borrowing $100,000 from a bank for 1 year at a stated interest rate of 9%. What is the
effective interest rate to Elan if this borrowing is in the form of a discounted note?

A. 8.19%
B. 9.89%
C. 9.81%
D. 9.00%

Question 19 - ICMA 1603.P2.016 - Short-term Bank Loans and Other S-T Financing

A commercial bank offered a $100,000 one-year loan with an annual interest rate of 6% and a 10% compensating
balance. What is the effective annual interest rate of this loan?

A. 7.00%.
B. 6.00%.
C. 6.67%.
D. 5.45%.

Question 20 - ICMA 10.P2.176 - Short-term Bank Loans and Other S-T Financing

Megatech Inc. is a large publicly-held firm. The treasurer is making an analysis of the short-term financing options
available for the third quarter, as the company will need an average of $8 million for the month of July, $12 million for
August, and $10 million for September. The(c) HOCK international,
following page 5
options are available.
I. Issue commercial paper on July 1 in an amount sufficient to net Megatech $12 million at an effective rate of 7% per
year. Any temporarily excess funds will be deposited in Megatech's investment account at First City Bank and earn
interest at an annual rate of 4%.
Part 2 : 01/14/19 11:29:25

Megatech Inc. is a large publicly-held firm. The treasurer is making an analysis of the short-term financing options
available for the third quarter, as the company will need an average of $8 million for the month of July, $12 million for
August, and $10 million for September. The following options are available.
I. Issue commercial paper on July 1 in an amount sufficient to net Megatech $12 million at an effective rate of 7% per
year. Any temporarily excess funds will be deposited in Megatech's investment account at First City Bank and earn
interest at an annual rate of 4%.
II.Utilize a line of credit from First City Bank with interest accruing monthly on the amount utilized at the prime rate,
which is estimated to be 8% in July and August and 8.5% in September.

Based on this information, which one of the following actions should the treasurer take?

A. Use the line of credit, since it is approximately $5,800 less expensive than issuing commercial paper.
B. Issue commercial paper, since it is approximately $35,000 less expensive than the line of credit.
C. Issue commercial paper, since it is approximately $14,200 less expensive than the line of credit.
D. Use the line of credit, since it is approximately $15,000 less expensive than issuing commercial paper.

Question 21 - ICMA 1603.P2.013 - Short-term Bank Loans and Other S-T Financing

A company had total sales of $500,000 in the first quarter of the year, which was the same amount as it recorded in the
first quarter of the prior year. However, its accounts receivable balance increased from $230,000 last year to $300,000
this year. Which one of the following is the most likely explanation for the increase in the accounts receivable balance?

A. The company shortened its payment terms in the current year from 60 days to 30 days.
B. The company initiated the use of factoring in the current year.
C. The company discontinued the use of factoring in the current year.
D. The company hired more people in its credit and collections department.

Question 22 - ICMA 10.P2.143 - Short-term Bank Loans and Other S-T Financing

Mandel Inc. has a zero-balance account with a commercial bank. The bank sweeps any excess cash into a
commercial investment account earning interest at the rate of 4% per year, payable monthly. When Mandel has a cash
deficit, a line of credit is used which has an interest rate of 8% per year, payable monthly based on the amount used.
Mandel expects to have a $2 million cash balance on January 1 of next year. Net cash flows for the first half of the
year, excluding the effects of interest received or paid, are forecasted (in millions of dollars) as follows.
Jan Feb Mar Apr May Jun
Net cash inflows (millions of dollars) +2 +1 −5 −3 −2 +6

Assuming all cash-flows occur at the end of each month, approximately how much interest will Mandel incur for this
period?

A. $53,000 net interest paid.


B. $195,000 net interest paid.
C. $76,000 net interest paid.
D. $16,000 net interest paid.

Question 23 - ICMA 10.P2.182 - Short-term Bank Loans and Other S-T Financing

Approximately what amount of compensating balance would be required for a stated interest rate of 10% to equal an
effective interest rate of 10.31% on a $100,000,000 one-year loan?

A. $3,100,000.
B. Not enough information is given. (c) HOCK international, page 6
C. $310,000.
D. $3,000,000.
Part 2 : 01/14/19 11:29:25

A. $3,100,000.
B. Not enough information is given.
C. $310,000.
D. $3,000,000.

Question 24 - ICMA 10.P2.185 - Short-term Bank Loans and Other S-T Financing

Frame Industries has arranged a revolving line of credit for the upcoming year with a commercial bank. The
arrangement is for $20 million, with interest payable monthly on the amount utilized at the bank's prime rate and an
annual commitment fee of one-half of 1 percent, computed and payable monthly on the unused portion of the line.
Frame estimates that the prime rate for the upcoming year will be 8%, and expects the following average amounts to
be borrowed by quarter.
Amount
Quarter Borrowed
First $10,000,000
Second 20,000,000
Third 20,000,000
Fourth 5,000,000

How much will Frame owe to the bank next year in interest and fees?

A. $1,168,750.
B. $1,131,250.
C. $1,118,750.
D. $1,200,000.

Question 25 - ICMA 10.P2.144 - Short-term Bank Loans and Other S-T Financing

Dexter Products receives $25,000 worth of merchandise from its major supplier on the 15th and 30th of each month.
The goods are sold on terms of 1/15, net 45, and Dexter has been paying on the net due date and foregoing the
discount. A local bank offered Dexter a loan at an interest rate of 10%. What will be the net annual savings to Dexter if
it borrows from the bank and utilizes the funds to take advantage of the trade discount?

A. $1,575.
B. $1,050.
C. $525.
D. $2,250.

Question 26 - CMA 1289 1.21 - Short-term Bank Loans and Other S-T Financing

The Altmane Corporation was recently quoted terms on a commercial bank loan of 7% discounted interest with a 20%
compensating balance. The term of the loan is 1 year. The effective cost of borrowing is (rounded to the nearest
hundredth)

A. 8.75%.
B. 9.59%.
C. 7.53%.
D. 9.41%.

(c) HOCK international, page 7


Question 27 - ICMA 10.P2.181 - Short-term Bank Loans and Other S-T Financing

Keller Products needs $150,000 of additional funds over the next year in order to satisfy a significant increase in
Part 2 : 01/14/19 11:29:25

Question 27 - ICMA 10.P2.181 - Short-term Bank Loans and Other S-T Financing

Keller Products needs $150,000 of additional funds over the next year in order to satisfy a significant increase in
demand. A commercial bank has offered Keller a one-year loan at a nominal rate of 8%, which requires a 15%
compensating balance. How much would Keller have to borrow, assuming it would need to cover the compensating
balance with the loan proceeds?

A. $172,500.
B. $176,471.
C. $194,805.
D. $130,435.

Question 28 - CMA 696 1.14 - Short-term Bank Loans and Other S-T Financing

Which one of the following responses is not an advantage to a corporation that uses the commercial paper market for
short-term financing?

A. The borrower avoids the expense of maintaining a compensating balance with a commercial bank.
B. There are no restrictions as to the type of corporation that can enter into this market.
C. This market provides a broad distribution for borrowing.
D. This market provides more funds at lower rates than other methods provide.

Question 29 - CIA 594 IV.51 - Short-term Bank Loans and Other S-T Financing

A manufacturing firm wants to obtain a short term loan and has approached several lending institutions. All of the
potential lenders are offering the same nominal interest rate but the terms of the loans vary. Which of the following
combinations of loan terms will be most attractive for the borrowing firm?

A. Discount interest, no compensating balance.


B. Simple interest, no compensating balance.
C. Discount interest, 20% compensating balance required.
D. Simple interest, 20% compensating balance required.

Question 30 - ICMA 10.P2.180 - Short-term Bank Loans and Other S-T Financing

Gates Inc. has been offered a one-year loan by its commercial bank. The instrument is a discounted note with a stated
interest rate of 9%. If Gates needs $300,000 for use in the business, what should the face value of the note be?

A. $329,670.
B. $327,000.
C. $275,229.
D. $327,154.

Question 31 - ICMA 10.P2.179 - Short-term Bank Loans and Other S-T Financing

Lang National Bank offered a one-year loan(c) HOCK


to a international,
commercial page
customer. The8instrument is a discounted note with a
nominal rate of 12%. What is the effective interest rate to the borrower?

A. 10.71%
B. 13.64%
Part 2 : 01/14/19 11:29:25

Question 31 - ICMA 10.P2.179 - Short-term Bank Loans and Other S-T Financing

Lang National Bank offered a one-year loan to a commercial customer. The instrument is a discounted note with a
nominal rate of 12%. What is the effective interest rate to the borrower?

A. 10.71%
B. 13.64%
C. 13.20%
D. 12.00%

Question 32 - CMA 697 1.15 - Short-term Bank Loans and Other S-T Financing

The treasury analyst for Garth Manufacturing has estimated the cash flows for the first half of next year (ignoring any
short-term borrowings) as follows.
Cash (millions)
Inflows Outflows
January $2 $1
February 2 4
March 2 5
April 2 3
May 4 2
June 5 3

Garth has a line of credit of up to $4 million on which it pays interest monthly at a rate of 1% of the amount utilized.
Garth is expected to have a cash balance of $2 million on January 1 and no amount utilized on its line of credit.
Assuming all cash flows occur at the end of the month, approximately how much will Garth pay in interest during the
first half of the year?

A. $132,000
B. Zero.
C. $80,000
D. $61,000

Question 33 - ICMA 10.P2.174 - Short-term Bank Loans and Other S-T Financing

A manufacturer with seasonal sales would be most likely to obtain which one of the following types of loans from a
commercial bank to finance the need for a fixed amount of additional working capital during the busy season?

A. Unsecured short-term term loan.


B. Installment loan.
C. Insurance company term loan.
D. Transaction loan.

Question 34 - CMA 697 1.19 - Short-term Bank Loans and Other S-T Financing

Hagar Company's bank requires a compensating balance of 20% on a $100,000 loan. If the stated interest on the loan
is 7%, what is the effective cost of the loan?

A. 7.00%
B. 8.40%
C. 5.83%
(c) HOCK international, page 9
D. 8.75%
Part 2 : 01/14/19 11:29:25

B. 8.40%
C. 5.83%
D. 8.75%

Question 1 - CMA 1289 1.22 - Short-term Bank Loans and Other S-T Financing

A. Commercial paper is an unsecured note that is able to be issued only by the largest, most credit-worthy companies.
Therefore, it is not a source of financing for most companies.

B. Commercial paper is an unsecured note that is issued only by the largest, most stable and credit-worthy
companies. Commercial paper is sold at a discount and usually is very short-term. Usually, commercial paper
is cheaper than a bank loan because the nature of the companies that are able to issue commercial paper
greatly reduces the risk to the investor.

C. Generally a commission is charged on commercial paper just as on other instruments.

D. Commercial paper is an unsecured note so there is no collateral available to the investor in the case of default by
the issuer.

Question 2 - ICMA 10.P2.183 - Short-term Bank Loans and Other S-T Financing

A.

This is 7% × 1.20. This is not the correct way to find the answer to this question.

The correct way to calculate the effective annual rate on this loan is to first calculate the annual interest that will be due
on $100,000 at a stated rate of 7% for one year. Then, find the net funds from the loan the borrower will have available
after deducting the compensating balance requirement. The effective annual interest rate on a loan with a
compensating balance when no interest is received on the compensating balance is the annual interest due divided by
the net funds the borrower will have available to use.

B. 7.0% is the stated interest rate on the loan. Because the borrower has a compensating balance requirement, not all
of the loan proceeds will be available to the borrower to use. The effective annual interest rate on a loan with a
compensating balance when no interest is received on the compensating balance is the annual interest due divided by
the net funds the borrower will have available to use, and it is different from the stated interest rate.

C.

This is 20% minus 7%. That is not the correct way to find the answer to this question.

The correct way to calculate the effective annual rate on this loan is to first calculate the annual interest that will be due
on $100,000 at a stated rate of 7% for one year. Then, find the net funds from the loan the borrower will have available
after deducting the compensating balance requirement. The effective annual interest rate on a loan with a
compensating balance when no interest is received on the compensating balance is the annual interest due divided by
the net funds the borrower will have available to use.

D. The annual interest on $100,000 at a stated interest rate of 7% is $7,000 ($100,000 × 0.07). The net available
funds to the borrower if a 20% compensating balance is required is 80% of $100,000, or $80,000. The effective
annual interest rate on a loan with a compensating balance when no interest is received on the compensating
balance is the annual interest due divided by the net funds the borrower will have available to use. Thus the
effective annual interest rate on the loan is $7,000 ÷ $80,000, which equals 0.0875 or 8.75%.

(c) HOCK international, page 10


Question 3 - ICMA 10.P2.150 - Short-term Bank Loans and Other S-T Financing

A.
Part 2 : 01/14/19 11:29:25

Question 1 - CMA 1289 1.22 - Short-term Bank Loans and Other S-T Financing

A. Commercial paper is an unsecured note that is able to be issued only by the largest, most credit-worthy companies.
Therefore, it is not a source of financing for most companies.

B. Commercial paper is an unsecured note that is issued only by the largest, most stable and credit-worthy
companies. Commercial paper is sold at a discount and usually is very short-term. Usually, commercial paper
is cheaper than a bank loan because the nature of the companies that are able to issue commercial paper
greatly reduces the risk to the investor.

C. Generally a commission is charged on commercial paper just as on other instruments.

D. Commercial paper is an unsecured note so there is no collateral available to the investor in the case of default by
the issuer.

Question 2 - ICMA 10.P2.183 - Short-term Bank Loans and Other S-T Financing

A.

This is 7% × 1.20. This is not the correct way to find the answer to this question.

The correct way to calculate the effective annual rate on this loan is to first calculate the annual interest that will be due
on $100,000 at a stated rate of 7% for one year. Then, find the net funds from the loan the borrower will have available
after deducting the compensating balance requirement. The effective annual interest rate on a loan with a
compensating balance when no interest is received on the compensating balance is the annual interest due divided by
the net funds the borrower will have available to use.

B. 7.0% is the stated interest rate on the loan. Because the borrower has a compensating balance requirement, not all
of the loan proceeds will be available to the borrower to use. The effective annual interest rate on a loan with a
compensating balance when no interest is received on the compensating balance is the annual interest due divided by
the net funds the borrower will have available to use, and it is different from the stated interest rate.

C.

This is 20% minus 7%. That is not the correct way to find the answer to this question.

The correct way to calculate the effective annual rate on this loan is to first calculate the annual interest that will be due
on $100,000 at a stated rate of 7% for one year. Then, find the net funds from the loan the borrower will have available
after deducting the compensating balance requirement. The effective annual interest rate on a loan with a
compensating balance when no interest is received on the compensating balance is the annual interest due divided by
the net funds the borrower will have available to use.

D. The annual interest on $100,000 at a stated interest rate of 7% is $7,000 ($100,000 × 0.07). The net available
funds to the borrower if a 20% compensating balance is required is 80% of $100,000, or $80,000. The effective
annual interest rate on a loan with a compensating balance when no interest is received on the compensating
balance is the annual interest due divided by the net funds the borrower will have available to use. Thus the
effective annual interest rate on the loan is $7,000 ÷ $80,000, which equals 0.0875 or 8.75%.

Question 3 - ICMA 10.P2.150 - Short-term Bank Loans and Other S-T Financing

A.

The way to solve this is to calculate the effective annual percentage rate of each of the three types of
borrowing. (c) HOCK international, page 11

Commercial paper: The commercial paper is issued quarterly. The question does not state when each
commercial paper issuance is due, so we must assume that each one will be paid off by the next one, that is,
Part 2 : 01/14/19 11:29:25

The way to solve this is to calculate the effective annual percentage rate of each of the three types of
borrowing.

Commercial paper: The commercial paper is issued quarterly. The question does not state when each
commercial paper issuance is due, so we must assume that each one will be paid off by the next one, that is,
in 3 months. Therefore, the amount outstanding for one year will be a continuous amount of $8,800,000. The
amount of interest that will be payable each quarter for that outstanding balance will be the difference
between $9.1 million and $8.8 million, which is $300,000. To convert that quarterly interest payment to an
annual interest amount, we multiply it by 4. The annual interest due will thus be $1,200,000. So the annual
percentage rate for the commercial paper will be $1,200,000 ÷ $8,800,000, which is 13.64%.

Bank: This rate is given as 12%.

Suppliers: The formula to calculate the cost of not taking a supplier’s discount is:
360 Discount %
×
Total period for payment − Period for discount payment 100% − Discount %

Duoplan's cost of trade credit is:


360 0.02
× =
90 − 30 1.00 − 0.02

6 × 0.020408 = 0.1224 or 12.24%.

In comparing the cost of any source of funding with the cost of not taking a discount, the assumption is that
the company would use the borrowed funds to take the discount on the trade credit, thereby avoiding the high
cost of not taking the discount. If the company does not do that, then it will end up paying interest in both
places.

So assuming Duoplan will borrow from the bank and use the borrowed funds to take the suppliers' discounts,
the bank’s rate is the lowest of the three.

B. The rates on the three options are different enough that, based on the outstanding borrowings, the difference in the
interest costs will be significant.

C. The commercial paper is issued quarterly. The question does not state when each commercial paper issuance is
due, so we must assume that each one will be paid off by the next one, that is, in 3 months. Therefore, the amount
outstanding for one year will be a continuous amount of $8,800,000. The amount of interest that will be payable each
quarter for that outstanding balance will be the difference between $9.1 million and $8.8 million, which is $300,000. To
convert that quarterly interest payment to an annual interest amount, we multiply it by 4. The annual interest due will
thus be $1,200,000. So the annual percentage rate for the commercial paper will be $1,200,000 ÷ $8,800,000, which is
13.64%. This is not the lowest available cost of funds.

D.

The formula to calculate the cost of not taking a supplier's discount is:
360 Discount %
×
Total period for payment − Period for discount payment 100% − Discount %

Duoplan's cost of trade credit is:


360 0.02
× =
90 − 30 1.00 − 0.02

6 × 0.020408 = 0.1224 or 12.24%.

This is not the lowest available cost of funds.

(c) HOCK international, page 12


Question 4 - CMA Sample Q1.7 - Short-term Bank Loans and Other S-T Financing

A. In a loan with a compensating balance the borrower does not actually receive all of the monies that are
loaned because they must keep on deposit with the bank some of the money as the compensating balance.
Part 2 : 01/14/19 11:29:25

Question 4 - CMA Sample Q1.7 - Short-term Bank Loans and Other S-T Financing

A. In a loan with a compensating balance the borrower does not actually receive all of the monies that are
loaned because they must keep on deposit with the bank some of the money as the compensating balance.
However, they must pay interest on the full amount of the loan. Therefore, Scott will pay $36,000 in interest but
will receive only $255,000. This makes the effective interest rate 14.12% ($36,000 ÷ $255,000).

B. In a loan with a compensating balance the borrower does not actually receive all of the monies that are loaned
because they must keep on deposit with the bank some of the money as the compensating balance. However, they
must pay interest on the full amount of the loan. Because of this the effective interest rate on the loan is higher than the
stated (or nominal) rate of the loan. This rate is lower. See the correct answer for a complete explanation.

C. This is the stated interest rate on the loan. In a loan with a compensating balance the borrower does not actually
receive all of the monies that are loaned because they must keep on deposit with the bank some of the money as the
compensating balance. However, they must pay interest on the full amount of the loan. Because of this the effective
interest rate on the loan is higher than the stated (or nominal) rate of the loan. See the correct answer for a complete
explanation.

D. This answer takes into account only 11 months of interest. However, the question asks for the effective annual rate.
The effective annual rate assumes that the balance that was outstanding on the loan for part of the year was
outstanding for the full year, so we need to use 12 months of interest. See the correct answer for a complete
explanation.

Question 5 - ICMA 10.P2.171 - Short-term Bank Loans and Other S-T Financing

A.

During April and May, no amounts will be drawn on the line, so the company will owe $12,500 (0.0025 ×
$5,000,000) for the full unused committed amount for each of those two months, or $12,500 × 2 = $25,000.

On April 1, the company will begin the quarter with $2,000,000 in cash. Net April inflows will be $2,000,000,
minus the April commitment fee of $5,000,000 × 0.0025, or $12,500, so the ending cash balance April 30 will be
$3,987,500 ($2,000,000 + $2,000,000 − $12,500).

Net May outflows will equal $7,000,000, but remember that these will not occur until the end of the month.
Therefore, the company will still have nothing outstanding on its line until May 31 and will again owe a
commitment fee for May equal to $5,000,000 × 0.0025, or $12,500. So net outflows for May will be $7,000,000 +
$12,500, or $7,012,500. That will create a negative cash balance of $3,987,500 − $7,012,500, or $(3,025,000), and
the company will borrow that amount on its line of credit on May 31.

Net June inflows will equal $4,000,000. At the end of June, the company will owe 9% interest on the amount
outstanding on the line ($3,025,000) for one month; plus the commitment fee of 0.0025 multiplied by the
unused portion of the line. The unused portion of the line will be $5,000,000 − $3,025,000, or $1,975,000.
Therefore, the commitment fee on the unused amount will be $1,975,000 × 0.0025, or $4,937.50. Interest for one
month is calculated by multiplying the outstanding amount of the loan by 9% (to get one year’s interest) and
then dividing that by 12 (to convert that to one month’s interest). Interest due for one month will be $3,025,000
× 0.09 ÷ 12, or $22,687.50. So the total fees paid to the bank at the end of June for the month of June will be
$4,937.50 + $22,687.50, or $27,625.

Therefore, the total amount paid to the bank during the quarter will be $12,500 + $12,500 + $27,625, or
$52,625. The question asks approximately how much will Burke pay to the bank. $52,625 is approximately
$52,600.

B.
(c) HOCK
This is not the correct answer. Please see the correct international, page 13explanation.
answer for a complete

We have been unable to determine how to calculate this incorrect answer choice. If you have calculated it, please let
us know how you did it so we can create a full explanation of why this answer choice is incorrect. Please send us an
Part 2 : 01/14/19 11:29:25

B.

This is not the correct answer. Please see the correct answer for a complete explanation.

We have been unable to determine how to calculate this incorrect answer choice. If you have calculated it, please let
us know how you did it so we can create a full explanation of why this answer choice is incorrect. Please send us an
email at support@hockinternational.com. Include the full Question ID number and the actual incorrect answer choice --
not its letter, because that can change with every study session created. The Question ID number appears in the upper
right corner of the ExamSuccess screen. Thank you in advance for helping us to make your HOCK study materials
better.

C.

This is not the correct answer. Please see the correct answer for a complete explanation.

We have been unable to determine how to calculate this incorrect answer choice. If you have calculated it, please let
us know how you did it so we can create a full explanation of why this answer choice is incorrect. Please send us an
email at support@hockinternational.com. Include the full Question ID number and the actual incorrect answer choice --
not its letter, because that can change with every study session created. The Question ID number appears at the top of
the question. Thank you in advance for helping us to make your HOCK study materials better.

D. This answer results from assuming that all cash flows occur at the beginning of each month. However, the question
says to assume that all cash flows occur at the end of the month.

Question 6 - CMA 694 1.21 - Short-term Bank Loans and Other S-T Financing

A. This answer results from dividing net interest of $37,000 by an available loan balance of $450,000. However, the net
interest of $37,000 results from assuming that 3% will be earned on the compensating deposit in the amount of
$100,000. The question says that the company ordinarily maintains a balance of $50,000 in its account for transaction
purposes. Therefore, only $50,000 of the loan proceeds will be added to the checking account, and the incremental
amount of interest earned will be 3% of $50,000.

B.

This answer results from assuming that $100,000 of the loan proceeds will be unavailable to the company because it
will be required as the compensating balance. However, the question says that the company ordinarily maintains a
balance of $50,000 in its account for transaction purposes. Therefore, only $50,000 of the loan proceeds will be added
to the checking account and will be unavailable to the company.

This will affect the calculation of the net interest expense (because the interest earned on the deposit will be based on
the $50,000 added to the account, not the full $100,000 requirement) as well as the available funds from the loan.

C. Since there is a compensating balance in the loan, the effective interest rate must be higher than the stated rate of
8%. Therefore, this answer is not possible. See the correct answer for a complete explanation.

D. In order to calculate the effective interest rate we need to determine how much the company will pay for
interest on the loan and how much they will receive from the bank in interest on their deposit. The amount of
the loan is $500,000, but with a compensating balance requirement of $100,000, they will not receive this entire
amount. The compensating balance is $100,000, but since they already have $50,000 in the bank, they need to
add only $50,000 of the loan proceeds to the balance to reach the required level. They will receive, then, in
essence, $450,000 from the bank. They will need to pay interest, however, on the entire $500,000. At 8% this is
$40,000 per year. However, they will also earn 3% on the $50,000 they added to their bank balance. This $1,500
is offset against the interest paid, giving a total interest expense of $38,500. So, the effective interest rate is
8.56% ($38,500 ÷ $450,000).

(c) HOCK international, page 14


Question 7 - CMA 1290 1.28 - Short-term Bank Loans and Other S-T Financing

A. This answer does not include the $1,200 of transaction costs, which must be included as a cost to Corbin. See the
correct answer for a complete explanation.
Part 2 : 01/14/19 11:29:25

Question 7 - CMA 1290 1.28 - Short-term Bank Loans and Other S-T Financing

A. This answer does not include the $1,200 of transaction costs, which must be included as a cost to Corbin. See the
correct answer for a complete explanation.

B.

The effective annual interest rate on a security or loan is calculated by calculating the cost for the holding
period (including discounted interest or other costs), annualizing it, and dividing the annualized cost by the
average balance outstanding for the period, which in this case is the amount of cash actually received.

The cost for 90 days will be $21,200, consisting of the $20,000 discount that is given to sell the paper and the
$1,200 transaction cost. Corbin will receive only $978,800 from this issuance, so the effective cost for 90 days
is $21,200. However, this is the cost for only 90 days, so it must be annualized by dividing it by the number of
days (90) to find the cost per day and multiplying the cost per day by 360 to find the cost for a full year.
$21,200 ÷ 90 × 360 = $84,800.

The annual cost is then divided by the average balance outstanding, which in this case is $978,800. $84,800 ÷
$978,800 = 0.0866 or 8.66%.

C. This answer does not include the $1,200 of transaction costs, which must be included as a cost to Corbin and it
considers that all of the $1,000,000 face amount was received by Corbin. See the correct answer for a complete
explanation.

D. This is the stated rate for three months. See the correct answer for a complete explanation.

Question 8 - CMA 1296 1.17 - Short-term Bank Loans and Other S-T Financing

A.

This is the amount of interest paid for one year multiplied by 2, the product divided by the net proceeds of the loan. The
effective annual rate is the interest paid for one year divided by the net proceeds of the loan that is outstanding during
the year.

B.

This is the discount rate multiplied by 2. When a loan is discounted, the bank withholds the amount of the interest from
the proceeds that are loaned. But when the loan becomes due the borrower needs to repay the full face amount of the
loan. The difference is the interest. The effective annual rate is the interest paid for one year divided by the net
proceeds of the loan that is outstanding during the year.

C. When a loan is discounted, the bank withholds the amount of the interest from the proceeds that are
loaned. But when the loan becomes due the borrower needs to repay the full face amount of the loan. In this
loan, the interest is $25,000. This is withheld from the proceeds of the loan and Frame will receive only
$100,000. The amount they will need to repay is $125,000, which includes $25,000 in interest. Given the
proceeds of $100,000, the effective interest rate is 25%.

D.

This is the nominal rate of interest. When there is discounted interest the effective rate of interest is higher than the
nominal rate of interest, so this answer is not possible.

When a loan is discounted, the bank withholds the amount of the interest from the proceeds that are loaned. But when
the loan becomes due the borrower needs to repay the full face amount of the loan. The difference is the interest. The
effective annual rate is the interest paid for one year divided by the net proceeds of the loan that is outstanding during
the year. (c) HOCK international, page 15
Part 2 : 01/14/19 11:29:25

the loan becomes due the borrower needs to repay the full face amount of the loan. The difference is the interest. The
effective annual rate is the interest paid for one year divided by the net proceeds of the loan that is outstanding during
the year.

Question 9 - CMA 1294 1.23 - Short-term Bank Loans and Other S-T Financing

A.

This answer results from multiplying the stated interest rate of 10% by the loan balance plus the compensating balance
and dividing the result by the amount of funds available to the borrower. The amount of interest due is based on the full
amount of the loan but not on the full amount of the loan plus the compensating balance.

B. This is the nominal rate of interest on the loan, not the effective interest rate. See the correct answer for a complete
explanation.

C. A compensating balance increases the effective interest rate of a loan because not all of the borrowed
money is actually available to the borrower. In a compensating balance agreement, the amount of the
compensating balance is withheld from the monies given to the borrower. However, the borrower must pay
interest on the full amount of the loan, including the compensating balance. So in this case, the borrower will
pay $50,000 in interest per year ($500,000 × 10%). However, because of the $50,000 compensating balance,
they will receive only $450,000. This makes the effective interest rate 11.1% ($50,000 ÷ $450,000).

D.

A compensating balance increases the effective interest rate of a loan because not all of the borrowed money is
actually available to the borrower. This rate is lower than the stated rate of 10%. It results from adding the amount of
the compensating balance to the loan proceeds instead of subtracting it.

Question 10 - CMA 1295 1.10 - Short-term Bank Loans and Other S-T Financing

A. This answer is the nominal rate of interest, not the effective rate of interest. See the correct answer for a complete
explanation.

B. This answer results from dividing $19,200 by $300,000, the gross amount of the loan. $19,200 is 8% of $240,000,
the net amount of usable funds after the 20% compensating balance is subtracted. This is incorrect for two reasons:
(1) The numerator should be 8% of the gross amount of $300,000. And (2) the denominator should be the net amount
of usable funds, not the gross amount of the loan.

C. When there is a compensating balance, interest is paid on the entire amount of the loan, but not all of the
loan amount is available to the borrower. The interest paid is 8% of $300,000, or $24,000. Because of the
compensating balance, only $240,000 of loan proceeds are available to use. Dividing the interest to be paid of
$24,000 by the available funds gives us the effective interest rate of 10%.

D. This is the percentage of the loan amount that must be kept in a compensating balance account and is not available
to use. It is not the effective rate of interest on the loan.

Question 11 - CMA 696 1.11 - Short-term Bank Loans and Other S-T Financing

A. This is $15,000 interest per year on the loan divided by the net usable loan proceeds of $225,000. This answer fails
to take into account the interest that will be earned on the money deposited to meet the compensating balance
requirement. See the correct answer for a complete explanation.
(c) HOCK international, page 16
B. This answer fails to take into consideration the fact that the company ordinarily maintains a balance of $25,000 in its
checking account for transaction purposes. Thus, the amount of the loan proceeds that will be added to the checking
account needs to be only $25,000, not the full $50,000 required.
Part 2 : 01/14/19 11:29:25

to take into account the interest that will be earned on the money deposited to meet the compensating balance
requirement. See the correct answer for a complete explanation.

B. This answer fails to take into consideration the fact that the company ordinarily maintains a balance of $25,000 in its
checking account for transaction purposes. Thus, the amount of the loan proceeds that will be added to the checking
account needs to be only $25,000, not the full $50,000 required.

C.

In a loan with a compensating balance, the borrower pays interest on the full amount of the loan but does not
receive the use of the full amount of the loan in cash, since they are required to leave some of it on deposit as
a compensating balance.

In this case, since they already maintain a $25,000 balance at the bank, they will need to add only $25,000 from
the loan proceeds to meet the compensating balance requirement. Therefore, the company will have the use
of $225,000 of the loan but they will pay interest of 6% on the full $250,000 loan amount. $250,000 × 0.06 equals
$15,000 of annual interest expense. However, this interest expense is reduced by the interest that will be
earned on the money that was deposited to meet the compensating balance requirement.

The incremental amount of the deposit increase is not the full $50,000 of the required compensating balance,
but only the $25,000 that the company needed to add to what was already in the bank. Interest earned on
additional $25,000 at 2% per annum equals $500 interest received. This interest received offsets the larger
interest cost, making a net interest expense of $14,500.

The effective interest rate on the loan is thus $14,500 ÷ the $225,000 received, or 6.44%.

D. This answer is incorrect because in a loan with a compensating balance the effective interest rate is higher than the
nominal interest rate. See the correct answer for a complete explanation.

Question 12 - CMA 1296 1.16 - Short-term Bank Loans and Other S-T Financing

A. When the company issues the commercial paper they will receive $100,000. Commercial paper is issued at
a discount. The face amount and the amount the company needs to repay is $110,000 so the difference
between that and the amount the receive is interest. However, the term of the commercial paper is only 6
months, so this needs to be done twice a year. Therefore, the annual cost of borrowing $100,000 will be
$20,000, so the effective annual interest rate is 20% ($20,000 divided by the $100,000 net proceeds available to
the company all year).

B. This is $10,000 (interest for 6 months) divided by $110,000. This is incorrect for two reasons. One, interest rates are
always annual interest rates, and this interest is for only 6 months. The interest for 6 months needs to be annualized by
multiplying it by 2. The interest for one full year will be 2 × $10,000. And two, the funds available will be only $100,000,
not $110,000. Commercial paper is sold at a discount. For each commercial paper issue, the company will receive
$100,000 and in 6 months it will have to repay $110,000, representing the principal received of $100,000 plus interest
for 6 months of $10,000. The annual interest rate is the total interest for one full year divided by the usable funds
received, assumed to be outstanding for one full year.

C. This is the effective rate for one of the issuances of commercial paper, but since it is only for 6 months, this needs
to be doubled.

D. This is $10,000 × 2 / $110,000. The cost in terms of an annual rate is the amount of interest paid during the course
of a year ($10,000 × 2) divided by the amount of cash the company receives from the issuance of the commercial
paper, assumed to be outstanding for a year. The amount of cash the company will receive from issuing the
commercial paper is only $100,000, not $110,000. (It will receive $100,000 and in 6 months it will have to repay
$110,000, representing the principal received of $100,000 plus interest for 6 months of $10,000; and then it will have to
issue the commercial paper again for another 6 months.) So to calculate the cost as an annual percentage, we need to
divide the interest paid for one full year by the amount of funds available for one full year from the two issuances.
(c) HOCK international, page 17

Question 13 - ICMA 10.P2.184 - Short-term Bank Loans and Other S-T Financing
Part 2 : 01/14/19 11:29:25

divide the interest paid for one full year by the amount of funds available for one full year from the two issuances.

Question 13 - ICMA 10.P2.184 - Short-term Bank Loans and Other S-T Financing

A.

The effective annual interest rate on a loan with a compensating balance when no interest is received on the
compensating balance is the annual interest due divided by the net funds the borrower will have available to use.

If the compensating balance on a $100,000,000 loan were $2,500,000, the net funds available to the borrower would
be $100,000,000& − $2,500,000, which is $97,500,000.

At a stated interest rate of 10%, as in Option #2, the annual interest due on the loan would be $10,000,000.

The effective annual interest rate would be $10,000,000 ÷ $97,500,000, which is 10.256% or rounded, 10.26%. The
requirement is that the effective annual interest rate be 10.25%. Although it is close, this answer choice does not fulfill
the requirement.

B.

The effective annual interest rate on a loan with a compensating balance when no interest is received on the
compensating balance is the annual interest due divided by the net funds the borrower will have available to use.

If the compensating balance on a $100,000,000 loan were $10,250,000, the net funds available to the borrower would
be $100,000,000 − $10,250,000, which is $89,750,000.

At a stated interest rate of 10%, as in Option #2, the annual interest due on the loan would be $10,000,000.

The effective annual interest rate would be $10,000,000 ÷ $89,750,000, which is 11.142% or rounded, 11.14%. The
requirement is that the effective annual interest rate be 10.25%. This answer choice does not fulfill the requirement.

C.

The effective annual interest rate on a loan with a compensating balance when no interest is received on the
compensating balance is the annual interest due divided by the net funds the borrower will have available to use.

If the compensating balance on a $100,000,000 loan were $250,000, the net funds available to the borrower would be
$100,000,000 − $250,000, which is $99,750,000.

At a stated interest rate of 10%, as in Option #2, the annual interest due on the loan would be $10,000,000.

The effective annual interest rate would be $10,000,000 ÷ $99,750,000, which is 10.025% or rounded, 10.03%. The
requirement is that the effective annual interest rate be 10.25%. This answer choice does not fulfill the requirement.

D.

The effective annual interest rate on a loan with a compensating balance when no interest is received on the
compensating balance is the annual interest due divided by the net funds the borrower will have available to
use.

Option #2 is a $100,000,000 loan with a stated interest rate of 10%. On a $100,000,000 loan at 10%, the interest
due for one year would be $10,000,000.

We next need to find the net funds available to the borrower that would cause the effective annual interest rate
to be 10.25% when the interest due for one year is $10,000,000. Once we have that, we can calculate what the
compensating balance requirement is.

Let X = the net funds available to the borrower.


(c) HOCK international, page 18

$10,000,000 ÷ X = 0.1025

X = $97,560,976
Part 2 : 01/14/19 11:29:25

compensating balance requirement is.

Let X = the net funds available to the borrower.

$10,000,000 ÷ X = 0.1025

X = $97,560,976

Therefore, the compensating balance required is $100,000,000 − $97,560,976, which equals $2,439,024.

$2,439,024.39 is not one of the answer choices given. The closest answer choice among those given is
$2,440,000, and the difference is simply a rounding difference. We can confirm that and that this is the correct
answer, as follows:

$100,000,000 − $2,440,000 = $97,560,000. Net available principal of $97,560,000 results in an effective annual
interest rate of 10.25% when rounded, as follows:

$10,000,000 ÷ $97,560,000 = 0.102501025 or 10.25%, rounded.

Question 14 - CMA 694 1.29 - Short-term Bank Loans and Other S-T Financing

A. This answer does not take into account the interest that is charged on the amount to be paid to the seller. See the
correct answer for a complete explanation.

B. The amount that is going to be received is going to be calculated by starting with the amount of the
receivable itself, $100,000. This will be reduced by the reserve and the commission, both of which are
calculated from the $100,000 amount. In total, this is 7% and reduces the amount to $93,000. This is the
amount on which the interest will be charged. The interest rate is 10% and this gives an annual interest of
$9,300. However, the time period before the receivable is due to be collected is only 3 months, so only 1/4 of
the annual interest will be charged. This is $2,325. This is also withheld from the amount received and brings
the amount received down to $90,675. If all of the receivables are actually collected when they become due,
the company will receive an additional $6,000 that was withheld as the reserve. In fact, any amount collected
in excess of $94,000 will be received by the company that sold the receivables.

C. This answer deducts an entire year of interest and ignores the reserve and commission. See the correct answer for
a complete explanation.

D. This answer charges an entire year's interest on the amount that is to be paid (loaned) to the seller of the
receivables. Because the receivables are only 180 day receivables, only one-quarter of the annual interest should be
deducted. See the correct answer for a complete explanation.

Question 15 - CMA 696 1.30 - Short-term Bank Loans and Other S-T Financing

A. This answer does not include the factor's fee. See the correct answer for a complete explanation.

B.

This is the net annual cost divided by the gross amount of receivables sold. The net annual cost should be divided by
the net amount of funding available to the company instead.

C. In the process of factoring the receivables, the company will save $18,000 per year in collection expenses.
However, there will also be some costs associated with this. Of the amount sold, the company will lose 2% of
the amount of receivables. This is $24,000 per year ($100,000 × 12 × 0.02), giving a net cost of $6,000 per year.
Additionally, they will need to pay 10% interest
(c) HOCK oninternational,
the amount that is 19
page advanced to them connected to the
factoring. They will receive 80% of the monthly amount of receivables, or $80,000 and because this will be
done each month we can treat this as if there is always $80,000 outstanding during the year. The annual
interest on this is $8,000 and adding this to the $6,000 net cost gives a total cost of $14,000 to receive a loan of
$80,000. Therefore the cost is 17.5% ($14,000 ÷ $80,000).
Part 2 : 01/14/19 11:29:25

the amount of receivables. This is $24,000 per year ($100,000 × 12 × 0.02), giving a net cost of $6,000 per year.
Additionally, they will need to pay 10% interest on the amount that is advanced to them connected to the
factoring. They will receive 80% of the monthly amount of receivables, or $80,000 and because this will be
done each month we can treat this as if there is always $80,000 outstanding during the year. The annual
interest on this is $8,000 and adding this to the $6,000 net cost gives a total cost of $14,000 to receive a loan of
$80,000. Therefore the cost is 17.5% ($14,000 ÷ $80,000).

D. This answer does not count the 2% for each month, but only for one month. See the correct answer for a complete
explanation.

Question 16 - CMA 1296 1.15 - Short-term Bank Loans and Other S-T Financing

A. When there is a compensating balance not all of the loaned funds are available to the borrower. The bank
retains the amount of the compensating balance, in this case $9,900 ($110,000 × 0.09). This means that the
borrower received only $100,100. However, the 12% interest is paid on the full $110,000 amount of the loan.
This is $13,200 for one year ($1000,000 × 0.09). When we divide this by the cash received from the loan, we get
the effective annual interest rate of 13.2%.

B.

This is the nominal rate of interest. When there is a compensating balance the effective rate of interest is higher than
the nominal rate of interest, so this answer is not possible.

When there is a compensating balance not all of the loaned funds are available to the borrower. The bank retains the
amount of the compensating balance. However, the interest is paid on the full amount of the loan. When we divide the
interest paid for one year by the cash received from the loan, we get the effective annual interest rate.

C.

This is the amount of the compensating balance requirement, not the effective annual interest rate.

When there is a compensating balance not all of the loaned funds are available to the borrower. The bank retains the
amount of the compensating balance. However, the interest is paid on the full amount of the loan. When we divide the
interest paid for one year by the cash received from the loan, we get the effective annual interest rate.

D.

This is the annual nominal rate of interest (12%) plus the compensating balance requirement (9%), not the effective
annual interest rate.

When there is a compensating balance not all of the loaned funds are available to the borrower. The bank retains the
amount of the compensating balance. However, the interest is paid on the full amount of the loan. When we divide the
interest paid for one year by the cash received from the loan, we get the effective annual interest rate.

Question 17 - ICMA 10.P2.186 - Short-term Bank Loans and Other S-T Financing

A. This is the stated interest rate plus the percentage of the total loan amount required to be maintained as a
compensating balance. This is not the correct way to calculate the effective annual interest rate on a loan with a
compensating balance requirement. The effective annual interest rate on a loan with a compensating balance when no
interest is received on the compensating balance is the annual interest due divided by the net funds the borrower will
have available to use.

B.

(c) HOCK
The effective annual interest rate on a loan with a international,
compensatingpage 20 when no interest is received on the
balance
compensating balance is the annual interest due divided by the net funds the borrower will have available to
use.
Part 2 : 01/14/19 11:29:25

B.

The effective annual interest rate on a loan with a compensating balance when no interest is received on the
compensating balance is the annual interest due divided by the net funds the borrower will have available to
use.

The annual interest due on a $100,000,000 loan with a stated interest rate of 8.00% is $8,000,000. If the
compensating balance requirement is $5,000,000, the net funds available to the borrower to use will be
$95,000,000. Thus, the effective annual interest rate will be $8,000,000 ÷ $95,000,000, which is 0.0842 or 8.42%.

C. 8% is the stated rate of interest. The effective annual interest rate on a loan with a compensating balance when no
interest is received on the compensating balance is the annual interest due divided by the net funds the borrower will
have available to use. That will be a different rate from the stated interest rate.

D. This is $8,000,000 divided by $105,000,000. The effective annual interest rate on a loan with a compensating
balance when no interest is received on the compensating balance is the annual interest due divided by the net funds
the borrower will have available to use. With a $5,000,000 compensating balance requirement on a $100,000,000 loan,
the borrower will have only $95,000,000 available to use.

Question 18 - CMA 1295 1.11 - Short-term Bank Loans and Other S-T Financing

A. This answer results from decreasing the loan amount by $9,000 in interest, then multiplying the remaining $91,000
by 9%, which equals $8,190. $8,190 divided by $100,000 is 8.19%. This is incorrect for two reasons: (1) The amount of
interest in the numerator should be $9,000, not $8,190. And (2) The denominator should be the net proceeds received
by the borrower, which is $91,000, not $100,000.

B. When interest is discounted, the amount that will be incurred as interest is withheld from the loan proceeds
given to the borrower. So, in this loan, the interest will be $9,000. However, instead of receiving $100,000 and
repaying $109,000, the bank withholds the interest and disburses only $91,000, while getting $100,000 when
the loan matures. Therefore, the effective rate is 9.89% since the interest expense is $9,000, but only $91,000
of funds were received.

C.

This is not the correct answer. Please see the correct answer for a complete explanation.

We have been unable to determine how to calculate this incorrect answer choice. If you have calculated it, please let
us know how you did it so we can create a full explanation of why this answer choice is incorrect. Please send us an
email at support@hockinternational.com. Include the full Question ID number and the actual incorrect answer choice --
not its letter, because that can change with every study session created. The Question ID number appears at the top of
the question. Thank you in advance for helping us to make your HOCK study materials better.

D. This is the nominal rate on the loan. When there is discounted interest, the effective rate is always higher than the
nominal rate, because the interest is withheld from the proceeds of the loan, and the borrower must repay the full face
value of the note, including the interest.

Question 19 - ICMA 1603.P2.016 - Short-term Bank Loans and Other S-T Financing

A.

This is not the correct answer. Please see the correct answer for an explanation.

We have been unable to determine how to calculate this incorrect answer choice. If you have calculated it, please let
us know how you did it so we can create a full explanation of why this answer choice is incorrect. Please send us an
(c) HOCK
email at support@hockinternational.com. Include international,
the full Question IDpage 21 actual incorrect answer choice—not its
and the
letter, because that can change with every study session created. The Question ID appears at the top of the question.
Thank you in advance for helping us to make your HOCK study materials better.

B. This answer results from not reducing the balance of the loan that is available to the borrower. Because of the
Part 2 : 01/14/19 11:29:25

We have been unable to determine how to calculate this incorrect answer choice. If you have calculated it, please let
us know how you did it so we can create a full explanation of why this answer choice is incorrect. Please send us an
email at support@hockinternational.com. Include the full Question ID and the actual incorrect answer choice—not its
letter, because that can change with every study session created. The Question ID appears at the top of the question.
Thank you in advance for helping us to make your HOCK study materials better.

B. This answer results from not reducing the balance of the loan that is available to the borrower. Because of the
compensating balance requirement, not all of the loan proceeds will be available to the borrower, but the borrower will
still be required to pay interest on 100% of the loan. The balance of the loan available to the borrower should be
reduced to reflect the amount of the compensating balance that the borrower will not be able to use.

C. The loan amount is $100,000 at 6% interest, so interest for one year will be $6,000. Because of the 10%
compensating balance requirement, the borrower has the use of only $90,000 of the loan proceeds. Therefore,
the borrower's effective annual interest rate on the loan is $6,000 ÷ $90,000, or 0.06667 which is 6.67%.

D. The effective annual interest rate of a loan with a compensating balance requirement will never be lower than the
nominal interest rate of the loan because the compensating balance requirement decreases the amount of loan funds
available to the borrower. Decreasing the amount of loan funds available to the borrower increases the effective annual
interest rate on the loan above the nominal amount because the borrower is paying interest on more funds than the
borrower has available.

Question 20 - ICMA 10.P2.176 - Short-term Bank Loans and Other S-T Financing

A. This answer results from not accounting for the interest income that can be earned by investing unneeded funds
from the issuance of the commercial paper.

B. This answer results from two mistakes: (1) using an outstanding balance of $12 million on the line of credit for the
full three months in calculating the interest expense charged on the line. The amount outstanding on the line each
month will be only the amount that Megatech needs that month, and those are the balances that interest will be
calculated on. (2) not accounting for the interest income that can be earned by investing unneeded funds from the
issuance of the commercial paper.

C.

Megatech will need $8 million in additional cash in July. In August, the company will need an additional $4
million in cash, for a total of $12 million needed. In September, the company's cash need will decrease to $10
million.

If the company utilizes commercial paper, it will receive $12 million at the beginning of July and will pay
interest on the full $12 million during July, August, and September; but during July and September, it will be
able to invest the amounts it does not need for operations: $4 million during July and $2 million during
September.

If the company utilizes the bank line of credit, it will pay interest on only the amounts outstanding, which will
be $8 million during July, $12 million during August, and $10 million during September.

The net cost of the commercial paper (interest cost less interest income) is:

Gross cost: $12,000,000 × 0.07 ÷ 12 months × 3 months = $210,000

Total income from investing unneeded funds (income amounts are in parentheses because they reduce the
cost):

July: $4,000,000 invested × 0.04 ÷ 12 months = $(13,333)


August: No unneeded funds to invest = 0
September: $2,000,000 invested × 0.04 ÷ 12 months = $(6,667)

Net cost of commercial paper = $210,000(c) HOCK international,


− $13,333 page 22
− $6,667 = $190,000

The cost of the line of credit is:


Part 2 : 01/14/19 11:29:25

September: $2,000,000 invested × 0.04 ÷ 12 months = $(6,667)

Net cost of commercial paper = $210,000 − $13,333 − $6,667 = $190,000

The cost of the line of credit is:

July: $8,000,000 borrowed × 0.08 ÷ 12 months = $53,333


August: $12,000,000 borrowed × 0.08 ÷ 12 = $80,000
September: $10,000,000 borrowed × 0.085 ÷ 12 = $70,833

Total cost of the line of credit = $53,333 + $80,000 + $70,833 = $204,166

Difference in cost:

$204,166 for the line of credit − $190,000 for the commercial paper = $14,166

The commercial paper is $14,166 less expensive than the cost of the line of credit, so the company should
issue commercial paper, since commercial paper is approximately $14,200 less expensive than the line of
credit.

D.

This is not the correct answer. Please see the correct answer for a complete explanation.

We have been unable to determine how to calculate this incorrect answer choice. If you have calculated it, please let
us know how you did it so we can create a full explanation of why this answer choice is incorrect. Please send us an
email at support@hockinternational.com. Include the full Question ID number and the actual incorrect answer choice --
not its letter, because that can change with every study session created. The Question ID number appears at the top of
the question. Thank you in advance for helping us to make your HOCK study materials better.

Question 21 - ICMA 1603.P2.013 - Short-term Bank Loans and Other S-T Financing

A. Shortening its payment terms would not cause the company's accounts receivable to increase.

B. If the company had initiated the use of factoring in the current year, the accounts receivable balance would be
expected to decline, not increase.

C. Discontinuing the use of factoring would cause the accounts receivable balance to increase because the
company would be carrying more of its own accounts receivable on its balance sheet instead of selling the
receivables.

D. Hiring more people in its credit and collections department would not cause the company's accounts receivable to
increase.

Question 22 - ICMA 10.P2.143 - Short-term Bank Loans and Other S-T Financing

A. This is approximately the amount of gross interest paid. However, it is not the net interest paid. The amount of
interest incurred/paid means the net amount of interest incurred, which is interest earned on invested funds minus
interest paid on borrowings.

B.

This is not the correct answer. Please see the correct answer for an explanation.

We have been unable to determine how to (c) HOCKthis


calculate international, page choice.
incorrect answer 23 If you have calculated it, please let
us know how you did it so we can create a full explanation of why this answer choice is incorrect. Please send us an
email at support@hockinternational.com. Include the full Question ID number and the actual incorrect answer choice --
not its letter, because that can change with every study session created. The Question ID number appears at the top of
Part 2 : 01/14/19 11:29:25

This is not the correct answer. Please see the correct answer for an explanation.

We have been unable to determine how to calculate this incorrect answer choice. If you have calculated it, please let
us know how you did it so we can create a full explanation of why this answer choice is incorrect. Please send us an
email at support@hockinternational.com. Include the full Question ID number and the actual incorrect answer choice --
not its letter, because that can change with every study session created. The Question ID number appears at the top of
the question. Thank you in advance for helping us to make your HOCK study materials better.

C.

This is not the correct answer. Please see the correct answer for an explanation.

We have been unable to determine how to calculate this incorrect answer choice. If you have calculated it, please let
us know how you did it so we can create a full explanation of why this answer choice is incorrect. Please send us an
email at support@hockinternational.com. Include the full Question ID number and the actual incorrect answer choice --
not its letter, because that can change with every study session created. The Question ID number appears at the top of
the question. Thank you in advance for helping us to make your HOCK study materials better.

D.

All cash flows are assumed to occur at the end of each month. Therefore, when money is borrowed on the line
of credit at month end or is outstanding on the line at the end of a month, the interest on it is not due until the
end of the following month, because then the loan will have been outstanding for one month’s time and will
have accrued one month’s interest. And when interest is earned, it is earned on invested funds that have been
invested for one month. Calculation of the ending investment or loan balance must include the amount of
interest the company will either be receiving or paying at the end of the month.

Here are the cash flows and the relevant interest earned on invested funds and paid on borrowings each
month.
Ending
Net Oper. Interest Earned Interest Incurred Investment
Date Cash Flow on Investment on Loan (Loan) Balance
Jan. 1 $2,000,000
(2,000,000 × 0.04/12)
Jan. 31 +2,000,000 =6,667 -0- 4,006,667
(4,006,667 × 0.04/12)
Feb. 28 +1,000,000 =13,356 -0- 5,020,023
(5,020,023 × 0.04/12)
Mar. 31 −5,000,000 =16,733 -0- 36,756
(36,756 × 0.04/12)
Apr. 30 −3,000,000 =123 -0- (2,963,121)
(2,963,121× 0.08/12)
May 31 −2,000,000 -0- =(19,754) (4,982,875)
(4,982,875 × 0.08/12)
June 30 +6,000,000 -0- =(33,219) 983,906
Totals 36,879 (52,973)

The amount of interest incurred/paid means the net amount of interest incurred, which is interest earned on
invested funds minus interest incurred/paid on borrowings. That is $36,879 interest earned minus $52,973
interest incurred/paid, which equals net interest incurred/paid of $(16,094). That is approximately $16,000.

Question 23 - ICMA 10.P2.182 - Short-term Bank Loans and Other S-T Financing

A. This answer results from dividing 10.31% of $100,000,000 by 10%. 10.31% of $100,000,000 is not the cash amount
of interest being charged on the loan. The cash amount of interest being charged on the loan is 10% of $100,000,000.
The correct answer to this question is the difference
(c) HOCK between the gross
international, pageamount
24 of the loan ($100,000,000) and the net
amount of the loan (the amount available after subtracting the compensating balance from the gross amount). The net
amount of the loan is the cash amount of interest being charged on the loan divided by 10.31%.

B. Enough information is given to answer the question.


Part 2 : 01/14/19 11:29:25

of interest being charged on the loan. The cash amount of interest being charged on the loan is 10% of $100,000,000.
The correct answer to this question is the difference between the gross amount of the loan ($100,000,000) and the net
amount of the loan (the amount available after subtracting the compensating balance from the gross amount). The net
amount of the loan is the cash amount of interest being charged on the loan divided by 10.31%.

B. Enough information is given to answer the question.

C.

This is not the correct answer. Please see the correct answer for a complete explanation.

We have been unable to determine how to calculate this incorrect answer choice. If you have calculated it, please let
us know how you did it so we can create a full explanation of why this answer choice is incorrect. Please send us an
email at support@hockinternational.com. Include the full Question ID number and the actual incorrect answer choice --
not its letter, because that can change with every study session created. The Question ID number appears at the top of
the question. Thank you in advance for helping us to make your HOCK study materials better.

D.

Annual Interest divided by Average Outstanding Principal = Effective Annual Interest Rate. Therefore, Average
Outstanding Principal × Effective Annual Interest Rate = Annual Interest. And also, Annual Interest ÷ Effective
Annual Interest Rate = Average Outstanding Principal.

Interest on a $100,000,000 loan at 10% for one year would be $10,000,000 ($100,000,000 × 0.10). We need to
find what the net proceeds of a $100,000,000 loan would be (after deducting the compensating balance) that
will result in an effective interest rate of 10.31%.

Since we know the interest amount ($10,000,000) and the rate (0.1031), we know everything for that formula
except the net principal amount. So let X represent the net principal.

10,000,000 / X = 0.1031.

Solving for X, we get X = $96,993,210. That is the net principal, after the compensating balance is subtracted,
because the amount of the compensating balance is not available to the borrower.

To find the amount of the compensating balance, we subtract the net principal from the gross loan amount of
$100,000,000. $100,000,000 − $96,993,210 = $3,006,790. The difference between that figure and this answer is
simply rounding. If you subtract $3,000,000 from $100,000,000, you get $97,000,000. $10,000,000 interest
(interest at 10% on $100,000,000) divided by a net principal amount of $97,000,000 is 10.309%, or rounded,
10.31%.

Question 24 - ICMA 10.P2.185 - Short-term Bank Loans and Other S-T Financing

A. This answer results from calculating a commitment fee of $18,750 for both the second quarter and the third quarter.
During the second and third quarters the line is completely drawn down, so there is no unused portion of the line and
no commitment fee is charged.

B.

The average loan balance outstanding that Frame expects to use during the upcoming year is ($10,000,000 +
$20,000,000 + $20,000,000 + $5,000,000) ÷ 4 = $13,750,000. The total interest due on the loan balance
outstanding during the year is therefore expected to be $13,750,000 × 0.08, or $1,100,000.

The commitment fee is due at an annual rate of one-half of 1 percent on the unused portion. The total
committed line amount is $20,000,000, and the planned average balance outstanding is $13,750,000. Therefore,
the average unused portion of the line is expected to be $20,000,000 − $13,750,000, or $6,250,000. The total
commitment fee on the unused balance for the year is thus expected to be $6,250,000 × 0.005, or $31,250.
(c) HOCK international, page 25
The total amount owed to the bank next year in interest and fees is therefore $1,100,000 + $31,250, or
$1,131,250.
Part 2 : 01/14/19 11:29:25

the average unused portion of the line is expected to be $20,000,000 − $13,750,000, or $6,250,000. The total
commitment fee on the unused balance for the year is thus expected to be $6,250,000 × 0.005, or $31,250.

The total amount owed to the bank next year in interest and fees is therefore $1,100,000 + $31,250, or
$1,131,250.

Note that since the same balance will not be outstanding on the line for each of the 4 quarters, the interest and
commitment fees due will not be the same for each quarter during the upcoming year. If quarterly amounts are
needed for planning purposes, the interest and commitment fee will need to be calculated separately for each
quarter using that quarter's planned outstanding balance and unused amount. However, quarterly amounts
are not required to answer this question.

C. This answer omits the commitment fee for the first quarter.

D. This answer results from calculating a commitment fee of $25,000 for each of the four quarters. A commitment fee
of $25,000 per quarter would be due only if the $20,000,000 line were unused all year, because the commitment fee is
due on the unused portion of the line only.

Question 25 - ICMA 10.P2.144 - Short-term Bank Loans and Other S-T Financing

A.

This is not the correct answer. Please see the correct answer for an explanation.

We have been unable to determine how to calculate this incorrect answer choice. If you have calculated it, please let
us know how you did it so we can create a full explanation of why this answer choice is incorrect. Please send us an
email at support@hockinternational.com. Include the full Question ID number and the actual incorrect answer choice --
not its letter, because that can change with every study session created. The Question ID number appears at the top of
the question. Thank you in advance for helping us to make your HOCK study materials better.

B.

The company can save 1% on each order by taking the discount. That is $25,000 × 0.01 = $250. If the company
borrows from a bank in order to make the payment on each order by the discount date (15 days after receipt
and 30 days before they usually pay), they will have to borrow $24,750 for each order ($25,000 − $250). To be
equivalent to the additional time they are now taking to pay for each order, each borrowing from the bank will
have to be for 30 days (45 days – 15 days).

Interest paid to the bank for each 30-day borrowing at the rate of 10% per year will be: $24,750 × 0.10 ÷ 360
days in a year × 30 days = $206.25.

The amount of savings for paying by day 15 is $250.

So the amount the company can save on each order by borrowing from the bank and paying by the discount
date (15 days) is the difference between the $250 discount and the bank’s interest charge for using $24,750 for
30 days, which is $206.25. The difference between $250 and $206.25 is $43.75 savings per order.

The company places 24 orders per year (2 per month). Therefore, the total savings per year is $43.75 × 24, or
$1,050 per year.

C.

This is the cost of not taking the discount minus the interest rate on the loan multiplied by $24,750, the amount the
company will pay for each order if it takes the discount. This is not the way to calculate the annual savings to Dexter if it
borrows from the bank and utilizes the funds to take advantage of the trade discount.

The difference between the amount saved on each order by taking the discount and the amount paid in interest for the
amount borrowed for each order multiplied (c) HOCK
by the international,
number page
of orders per 26is the total savings per year.
year

D.
Part 2 : 01/14/19 11:29:25

The difference between the amount saved on each order by taking the discount and the amount paid in interest for the
amount borrowed for each order multiplied by the number of orders per year is the total savings per year.

D.

This is not the correct answer. Please see the correct answer for an explanation.

We have been unable to determine how to calculate this incorrect answer choice. If you have calculated it, please let
us know how you did it so we can create a full explanation of why this answer choice is incorrect. Please send us an
email at support@hockinternational.com. Include the full Question ID number and the actual incorrect answer choice --
not its letter, because that can change with every study session created. The Question ID number appears at the top of
the question. Thank you in advance for helping us to make your HOCK study materials better.

Question 26 - CMA 1289 1.21 - Short-term Bank Loans and Other S-T Financing

A. This answer does not treat the interest as discounted interest. Because the interest is discounted, it should be
subtracted at the beginning of the loan to calculate the proceeds from the loan. See the correct answer for a complete
explanation.

B. In this question we have two different adjustments that need to be made in order to calculate the effective
interest rate. Also, in this question they do not give us an amount for the loan, so let us assume that the loan
is $100. The two adjustments are the 7% discounted interest and the compensating balance. Both of these will
reduce the proceeds actually received from the loan. The 7% interest is withheld at the inception of the loan
and this amounts to $7, reducing the proceeds to $93. However, the 20% compensating balance will also
reduce the proceeds. The compensating balance is calculated on the gross amount of the loan ($100), and
20% of $100 is $20. Therefore, the compensating balance plus the discount amount bring the proceeds down
to $73. The interest owed for one year is $7, but the proceeds are really only $73. This gives an effective rate of
9.59% ($7 ÷ $73).

C. This answer does not include the compensating balance, which reduces the amount of money received as proceeds
from the loan. See the correct answer for a complete explanation.

D.

This answer results from calculating the compensating balance by multiplying the discounted proceeds of the loan by
20%. Assuming a loan amount of $100, this answer results from dividing the interest ($7) by [($100 − $7) − ($93 ×
0.20)], or $7 ÷ $74.40, which equals 9.41%.

The compensating balance should be 20% of the gross loan balance, not 20% of the discounted loan balance. The
divisor should be [($100 − $7) − ($100 × 0.20)], or $73.

Question 27 - ICMA 10.P2.181 - Short-term Bank Loans and Other S-T Financing

A. This is $150,000 × 1.15. Multiplying by 1.15 effectively calculates 15% of $150,000 and adds that to $150,000.
However, $150,000 is how much Keller needs from the loan. The bank will calculate the amount of the compensating
balance requirement on the amount borrowed instead. The amount borrowed will need to include the compensating
balance requirement.

B.

Let X represent the amount of the loan and 0.15X represent the amount of the compensating deposit. We need
X (the amount of the loan) minus 0.15X (the amount of the compensating deposit) to be equal to $150,000. The
equation and its solution are:
(c) HOCK international, page 27
X − 0.15X = $150,000
0.85X = $150,000
X = $176,471
Part 2 : 01/14/19 11:29:25

X (the amount of the loan) minus 0.15X (the amount of the compensating deposit) to be equal to $150,000. The
equation and its solution are:

X − 0.15X = $150,000
0.85X = $150,000
X = $176,471

The problem could also be solved by dividing $150,000 by 0.85 (1.00 − 0.15). $150,000 ÷ 0.85 = $176,471.

C. This is $150,000 ÷ 0.77. This would be correct if this were a discounted note, discounted at its interest rate of 8%
and neither the interest amount nor the compensating balance required were available to the borrower. However, this
is not a discounted note. Only the compensating balance required will not be available to the borrower.

D.

This is $150,000 ÷ 1.15. The resulting amount is not a reasonable answer to the question.

A portion of the loan proceeds will not be available to Keller, because it will be used to cover the compensating deposit
requirement. Therefore, Keller will have to borrow more than $150,000 in order to have $150,000 available after
subtracting 15% of the loan amount. $130,435 is less than $150,000, so a loan of $130,435 amount could not possibly
provide the funds Keller needs and the funds to cover the compensating deposit required.

Question 28 - CMA 696 1.14 - Short-term Bank Loans and Other S-T Financing

A. Commercial paper requires no compensating balance, so this is an advantage of commercial paper.

B. Because commercial paper is a source of unsecured financing, only the most creditworthy companies can
issue commercial paper. Thus, there are restrictions as to the type of corporation that can enter the market.

C. The commercial paper market does provide a broad distribution for borrowing, so this is an advantage of
commercial paper.

D. Commercial paper does provide a lower rate than many other methods of financing, so this is an advantage of
commercial paper.

Question 29 - CIA 594 IV.51 - Short-term Bank Loans and Other S-T Financing

A. Discounted interest causes the effective interest rate to be higher than the stated rate for the loan. Therefore, a
discounted interest loan will not be more attractive for the borrowing firm.

B. Discounted interest and compensating balances both cause the effective interest rate to be higher than the
stated rate for the loan. Therefore, the loan that offers simple interest and no compensating balance will have
a lower effective rate than a loan with discounted interest and/or a compensating balance requirement.

C. Discounted interest and compensating balances both cause the effective interest rate to be higher than the stated
rate for the loan. Therefore, a loan with discounted interest and a compensating balance requirement will not be more
attractive for the borrowing firm.

D. A compensating balance causes the effective interest rate to be higher than the stated rate for the loan. Therefore,
a loan with a compensating balance requirement will not be more attractive for the borrowing firm.

(c)Bank
Question 30 - ICMA 10.P2.180 - Short-term HOCK international,
Loans page
and Other S-T 28
Financing

A.
Part 2 : 01/14/19 11:29:25

Question 30 - ICMA 10.P2.180 - Short-term Bank Loans and Other S-T Financing

A.

This is a discounted note, which means that the interest is calculated on the gross amount of the note, and
the interest is subtracted from the gross amount to find the amount the borrower actually receives. However,
the borrower must pay back the entire gross amount, including the interest.

If the discount rate is 9%, 9% of the gross loan amount will be subtracted from the face amount of the note to
determine the amount the borrower will receive. Gates needs $300,000. Therefore, the formula we will use to
find the face amount of the note will be:

X − 0.09X = $300,000

Where X = the face amount of the note

Simplifying that equation, we get:

0.91X = $300,000

Solving for X, we get X = $329,670; and that is the amount that Gates will need to borrow in order to receive a
net amount of $300,000 from the loan disbursement.

To prove that answer: $329,670 × 0.09 = $29,670.

$329,670 − $29,670 = $300,000, the net disbursement the borrower needs to receive.

B.

This is a discounted note, which means that the interest is calculated on the gross amount of the note, and the interest
is subtracted from the gross amount to find the amount the borrower actually receives. However, the borrower must
pay back the entire gross amount, including the interest.

This is $300,000 × 1.09. Multiplying by 1.09 effectively calculates 9% of $300,000 and adds that to $300,000. However,
$300,000 is what the net proceeds of the loan needs to be after the discounted interest has been subtracted from the
face amount of the note. The bank will calculate the 9% discounted interest on the face amount (the gross amount) of
the note instead.

C.

This is $300,000 ÷ 1.09. This is not a reasonable answer to the question.

This is a discounted note, which means that the interest is calculated on the gross amount of the note, and the interest
is subtracted from the gross amount to find the amount the borrower actually receives. However, the borrower must
pay back the entire gross amount, including the interest.

If the net proceeds of the note needs to be $300,000 and interest is subtracted from the face amount to calculate the
net proceeds, the face amount of the note must be greater than the $300,000 net proceeds needed. This amount is
less than the $300,000 net proceeds needed.

D.

This is $300,000 ÷ 0.917, the present value of $1 factor for 9% for one year.

The present value of a given amount (here unknown) is the amount an investor could invest today at the given interest
rate (here 9%) if the investor had it, and with the interest added, it would grow to the given amount in the given amount
of time (here one year). That is not the situation here.

This is a discounted note, which means that(c)the interest


HOCK is calculatedpage
international, on the
29gross amount of the note, and the interest
is subtracted from the gross amount to find the amount the borrower actually receives. However, the borrower must
pay back the entire gross amount, including the interest. The usual type of interest calculation will not work in this
situation, because of the fact that the interest amount paid by the borrower is not based on the amount the borrower
receives.
Part 2 : 01/14/19 11:29:25

This is a discounted note, which means that the interest is calculated on the gross amount of the note, and the interest
is subtracted from the gross amount to find the amount the borrower actually receives. However, the borrower must
pay back the entire gross amount, including the interest. The usual type of interest calculation will not work in this
situation, because of the fact that the interest amount paid by the borrower is not based on the amount the borrower
receives.

In fact, the effective rate of interest on a loan like this is higher than its stated rate because the borrower has to pay
interest on a principal amount that is greater than the amount the borrower receives. If the effective interest rate were
known, dividing $300,000 by the factor for that interest rate would result in the correct answer. But the effective interest
rate on the loan cannot be calculated until the correct answer to this question has been calculated and the interest
amount is known. So dividing $300,000 by the factor for the effective interest rate cannot be used to find the answer to
this problem, either.

Question 31 - ICMA 10.P2.179 - Short-term Bank Loans and Other S-T Financing

A. The easiest way to approach a question like this is to make up a gross amount for the loan and multiply it by the
nominal rate to calculate the amount of the discounted interest. Then subtract the amount of the discounted interest
from the gross amount of the loan to calculate the amount of the proceeds of the loan. Finally, divide the amount of the
annual interest by the amount of the proceeds of the loan. Since in this question the loan is for one year, the amount of
the annual interest is the same as the amount of the discounted interest. (If the loan were for a period other than one
year, it would be necessary to annualize the amount of the discounted interest.)

This answer results from adding the amount of the discounted interest to the gross amount for the loan to get the
amount of the proceeds for the loan, instead of subtracting it. Or, it may result from dividing 0.12 by (1 + the nominal
rate of 0.12).

B.

The easiest way to approach a question like this is to make up some numbers. So we will say that the loan
amount is $100,000. It is a discounted note, which means that the interest is subtracted from the proceeds of
the loan (the amount of money the borrower actually receives), and the borrower must pay back the entire
$100,000. So 12% of $100,000, or $12,000, will be subtracted from the proceeds, and the borrower will receive
$88,000. The borrower will be obligated to repay the full amount of the note, which will consist of the $88,000
loan proceeds plus the $12,000 interest, for a total of $100,000. Therefore, the effective interest rate will be
$12,000 ÷ $88,000 = 0.1364 or 13.64%.

Another way to calculate this answer is simply to divide 0.12 by (1 − 0.12):

0.12 ÷ 0.88 = 0.1364 or 13.64%.

C. This answer results from dividing 0.12 by the present value of $1 factor for 10% for 1 year. The easiest way to
approach a question like this is to make up a gross amount for the loan and multiply it by the nominal rate to calculate
the amount of the discounted interest. Then subtract the amount of the discounted interest from the gross amount of
the loan to calculate the amount of the proceeds of the loan. Finally, divide the amount of the annual interest by the
amount of the proceeds of the loan. Since in this question the loan is for one year, the amount of the annual interest is
the same as the amount of the discounted interest. (If the loan were for a period other than one year, it would be
necessary to annualize the amount of the discounted interest.)

D. 12% is the nominal interest rate on a discounted note. The question is asking for the effective interest rate. The
effective annual interest rate is the amount of interest paid for a year divided by the proceeds of the loan (the amount
the borrower actually receives). The amount the borrower actually receives will be the face amount of the note minus
the discounted interest amount.

Question 32 - CMA 697 1.15 - Short-term Bank


(c) HOCK international,
Loans and Other S-Tpage 30
Financing

A.

This is not the correct answer. Please see the correct answer for an explanation.
Part 2 : 01/14/19 11:29:25

Question 32 - CMA 697 1.15 - Short-term Bank Loans and Other S-T Financing

A.

This is not the correct answer. Please see the correct answer for an explanation.

We have been unable to determine how to calculate this incorrect answer choice. If you have calculated it, please let
us know how you did it so we can create a full explanation of why this answer choice is incorrect. Please send us an
email at support@hockinternational.com. Include the full Question ID number and the actual incorrect answer choice --
not its letter, because that can change with every study session created. The Question ID number appears at the top of
the question. Thank you in advance for helping us to make your HOCK study materials better.

B.

This answer could result from simply summing the beginning cash balance and the expected inflows and subtracting
the expected outflows. However, timing of the cash flows is important in determining the need for financing, and
financing will be needed, so interest expense will be greater than zero.

C.

This is not the correct answer. Please see the correct answer for an explanation.

We have been unable to determine how to calculate this incorrect answer choice. If you have calculated it, please let
us know how you did it so we can create a full explanation of why this answer choice is incorrect. Please send us an
email at support@hockinternational.com. Include the full Question ID number and the actual incorrect answer choice --
not its letter, because that can change with every study session created. The Question ID number appears at the top of
the question. Thank you in advance for helping us to make your HOCK study materials better.

D.

Garth will not need to borrow anything until the end of March because that is the first date when the cash
balance will be negative without borrowing.

At the end of March, Garth will need to borrow $2,000,000 to get its cash balance up to zero. That $2,000,000
will be outstanding during the month of April, since the question says that all cash flows occur at the end of
each month.

For the month of April, that $2,000,000 outstanding on the line of credit will incur $20,000 of interest, so that
interest will become an additional cash outflow at the end of April, making the required loan balance at the
end of April $3,020,000. Cash in for April is $2,000,000 and cash out for April is $3,000,000 + $20,000 interest;
so Garth will need to borrow another $1,000,000 as well as the amount due for interest. The ending cash
balance for April will be $0 + $2,000,000 cash inflow from operations − $3,000,000 cash outflow from
operations − $20,000 interest = $(1,020,000). So Garth will add that to its outstanding loan balance of
$2,000,000.

$3,020,000 will be outstanding on the loan during the month of May, and interest due at the end of May on the
outstanding balance will be $30,200.

For the month of May, cash inflow is $4,000,000 from operations and cash outflow is $2,000,000 from
operations plus the cash paid in interest of $30,200. Ending cash balance before any loans are repaid will be
$1,969,800. So the company will be able to pay down $1,969,800 on the line of credit at the end of May.

The amount of the loan outstanding at the end of May will be $1,050,200 ($3,020,000 − $1,969,800). So interest
accrued on the loan outstanding during the month of June will be $10,502.

In June, $5,000,000 of cash from operations comes in and only $3,000,000 cash from operations goes out, plus
the cash paid in interest of $10,502. At the end of the month, before any loan repayment is made, the company
will have $5,000,000 − $3,000,000 − $10,502, or $1,989,498. The company will be able to pay off the entire
principal outstanding on its line of credit(c)
ofHOCK
$1,050,200 at the end
international, of June
page 31 and will have $939,298 in cash left.
So the total interest paid for the first half of the year will be: $20,000 paid at the end of April + $30,200 paid at
the end of May + $10,502 paid at the end of June, for a total of $60,702. Since the question asks for the
approximate amount of interest paid during the first half of the year, $61,000 is the closest answer.
Part 2 : 01/14/19 11:29:25

will have $5,000,000 − $3,000,000 − $10,502, or $1,989,498. The company will be able to pay off the entire
principal outstanding on its line of credit of $1,050,200 at the end of June and will have $939,298 in cash left.

So the total interest paid for the first half of the year will be: $20,000 paid at the end of April + $30,200 paid at
the end of May + $10,502 paid at the end of June, for a total of $60,702. Since the question asks for the
approximate amount of interest paid during the first half of the year, $61,000 is the closest answer.

Question 33 - ICMA 10.P2.174 - Short-term Bank Loans and Other S-T Financing

A.

A short-term loan is a loan with a maturity date of less than one year in the future. A short-term loan, either
secured or unsecured, would be an appropriate type of loan to be used to finance the need for a fixed amount
of additional working capital during the busy season.

The collection of receivables from the selling season is the source of the repayment of the loan, so the loan
should be paid off about a month following the end of the company’s busy season. Therefore, the loan should
have a maturity date of about one month beyond the end of the company’s busy season, so the company will
have a chance to collect the receivables from the selling season to use to pay off the loan.

If the loan cannot be paid off on its maturity date, then something is wrong, because it means the company
has used the collection of the receivables for something other than to pay off the seasonal loan.

B.

An installment loan is a type of long-term financing, and it would have a maturity date of at least one year in the future.
Installment loans are generally used for the purchase of vehicles and other smaller fixed assets.

Financing that provides additional working capital to support the busy season is short-term financing, because it can be
repaid when the busy season is over and the receivables from the sales are collected. So an installment loan would not
be appropriate for this purpose.

C.

An insurance company term loan, or any term loan no matter who the lender is, would be used for long-term financing.
A term loan is a loan made to a business for long-term needs, such as purchase of fixed assets, that has a maturity
date of one year or more in the future.

Financing that provides additional working capital to support the busy season is short-term financing, because it can be
repaid when the busy season is over and the receivables from the sales are collected. So a term loan would not be
appropriate for this purpose.

D.

A transaction loan is a loan made for a specific purchase, such as a mortgage loan made for the purchase of real
estate or a term loan made for the purchase of equipment. Usually, the disbursement check is made out to the seller of
the item, so the lender can be certain that the loan is being used for its designated purpose.

A working capital loan such as the one mentioned in this problem would not be called a transaction loan, because it
would not be used to make just a single purchase from a single supplier. It would probably be used for multiple
inventory purchases from multiple suppliers, possibly over a period of several months. So it would not be practical for
the loan to be disbursed by means of checks made payable to each of the suppliers.

Question 34 - CMA 697 1.19 - Short-term Bank Loans and Other S-T Financing
(c) HOCK international, page 32
A. This is the stated interest rate on the loan. In a loan with a compensating balance the borrower does not actually
receive all of the monies that are loaned because they must keep on deposit with the bank some of the money as the
compensating balance. However, they must pay interest on the full amount of the loan. Because of this the effective
Part 2 : 01/14/19 11:29:25

Question 34 - CMA 697 1.19 - Short-term Bank Loans and Other S-T Financing

A. This is the stated interest rate on the loan. In a loan with a compensating balance the borrower does not actually
receive all of the monies that are loaned because they must keep on deposit with the bank some of the money as the
compensating balance. However, they must pay interest on the full amount of the loan. Because of this the effective
interest rate on the loan is higher than the stated (or nominal) rate of the loan. The interest payable divided by the
usable funds equals the effective annual rate.

B. This answer results from increasing the amount of interest due for one year by 20% and dividing the result by
$100,000. This is not the way a compensating balance requirement works. In a loan with a compensating balance
requirement, the borrower does not actually receive all of the monies that are loaned because they must keep on
deposit with the bank some of the money as the compensating balance. However, they must pay interest on the full
amount of the loan. The interest payable divided by the usable funds equals the effective annual rate.

C. This is the interest on $100,000 at 7% divided by the amount of the loan plus the compensating balance. In a loan
with a compensating balance the borrower does not actually receive all of the monies that are loaned because they
must keep on deposit with the bank some of the money as the compensating balance. However, they must pay interest
on the full amount of the loan. Because of this the effective interest rate on the loan is higher than the stated (or
nominal) rate of the loan. The interest payable divided by the usable funds equals the effective annual rate.

D. In a loan with a compensating balance the borrower does not actually receive all of the monies that are
loaned because they must keep on deposit with the bank some of the money as the compensating balance.
However, they must pay interest on the full amount of the loan. Therefore, Hagar will pay $7,000 in interest (7%
of $100,000) but will have use of only $80,000. The interest payable divided by the usable funds equals the
effective annual rate. The effective interest rate is 8.75% ($7,000 ÷ $80,000).

(c) HOCK international, page 33

Das könnte Ihnen auch gefallen